Những câu hỏi liên quan
trần duy anh
Xem chi tiết
Juki Mai
Xem chi tiết
Nguyễn Trần  Hoàng Hân
21 tháng 1 2017 lúc 8:51

mk ko biết mk mới học lớp nhỏ thôi . Đó là lớp này nè bn...... tự vào trang của mk coi đi nhé

Nguyễn Trần  Hoàng Hân
21 tháng 1 2017 lúc 8:52

hello

Nguyễn Chí Thành
9 tháng 11 2021 lúc 10:58

Aduf Lớp 8? Mh mới lớp 2 thui!!!!!

Khách vãng lai đã xóa
hoang gia kieu
Xem chi tiết

\(\frac{3}{2^2}.\frac{8}{3^2}.\frac{15}{4^2}.....\frac{899}{30^2}\)

\(=\frac{1.3}{2.2}.\frac{2.4}{3.3}.\frac{3.5}{4.4}.....\frac{29.31}{30.30}=\frac{1.2.3.....29}{2.3.4.....30}.\frac{3.4.5.....31}{2.3.4.....30}\)

\(=\frac{1}{2}.\frac{31}{30}=\frac{31}{60}\)

Xem chi tiết
Phạm Thị Thùy Linh
13 tháng 6 2019 lúc 20:40

\(A=x+\left(x+\frac{1}{5}\right)+\left(x+\frac{2}{5}\right)+\left(x+\frac{3}{5}\right)+\left(x+\frac{4}{5}\right)\)

\(=5x+\frac{1}{5}+\frac{2}{5}+\frac{3}{5}+\frac{4}{5}\)

\(=5x+2\)

\(B=5x\)

\(\Rightarrow A>B\)Với \(\forall\)\(x\)

T.Ps
13 tháng 6 2019 lúc 20:45

#)Giải :

\(A=\left[x\right]+\left[1+\frac{1}{5}\right]+\left[x+\frac{2}{5}\right]+\left[x+\frac{3}{5}\right]+\left[x+\frac{4}{5}\right]\)

Thay x = 3,7 vào biểu thức, ta có :

\(A=\left[3,7\right]+\left[3,7+\frac{1}{5}\right]+\left[3,7+\frac{2}{5}\right]+\left[3,7+\frac{3}{5}\right]+\left[3,7+\frac{4}{5}\right]\)

\(A=\left[3,7+3,7+3,7+3,7+3,7\right]+\left[1+\frac{1}{5}+\frac{2}{5}+\frac{3}{5}+\frac{4}{5}\right]\)

\(A=18,5+3\)

\(A=21,5\)

\(B=\left[5x\right]=\left[5\times3,7\right]=18,5\)

Vì 21,5 > 18,5 \(\Rightarrow A>B\)

zZz Cool Kid_new zZz
13 tháng 6 2019 lúc 20:51

Phạm Thị Thùy Linh+๖²⁴ʱŤ.Ƥεɳɠʉїɳş༉ ( Team TST 14 ):Cả 2 bạn đều nhầm chỗ  \(\left[a\right]\) rồi nha.\(\left[a\right]\) tức là phần nguyên của a nghĩa là số nguyên lớn nhất ko vượt quá a.

\(A=\left[x\right]+\left[x+\frac{1}{5}\right]+\left[x+\frac{2}{5}\right]+\left[x+\frac{3}{5}\right]+\left[x+\frac{4}{5}\right]\)

\(=\left[3,7\right]+\left[3,7+\frac{1}{5}\right]+\left[3,7+\frac{2}{5}\right]+\left[3,7+\frac{3}{5}\right]+\left[3,7+\frac{4}{5}\right]\)

\(=3+3+4+4+4\)

\(=18\)

\(B=\left[5x\right]\)

\(B=\left[18,5\right]\)

\(=18\)

Vậy \(A=B\left(=18\right)\)

❤Firei_Star❤
Xem chi tiết
Wall HaiAnh
1 tháng 5 2018 lúc 9:33

Ta có

\(A=\frac{\left(3\frac{2}{5}+\frac{1}{5}\right):2\frac{1}{2}}{\left(5\frac{3}{7}-2\frac{1}{4}\right):4\frac{43}{56}}\)                                                   \(B=\frac{1,2:\left(1\frac{1}{5}-1\frac{1}{4}\right)}{0,32+\frac{2}{25}}\)

\(\Leftrightarrow A=\frac{\left(\frac{17}{5}+\frac{1}{5}\right):\frac{5}{2}}{\left(\frac{38}{7}-\frac{9}{4}\right):\frac{276}{56}}\)                                            \(\Leftrightarrow B=\frac{\frac{6}{5}:\left(\frac{6}{5}-\frac{5}{4}\right)}{\frac{8}{25}+\frac{2}{25}}\)

\(\Leftrightarrow A=\frac{\frac{18}{5}:\frac{5}{2}}{\frac{89}{28}:\frac{276}{56}}\)                                                            \(\Leftrightarrow B=\frac{\frac{6}{5}:\left(-\frac{1}{20}\right)}{\frac{2}{5}}\)

\(\Leftrightarrow A=\frac{\frac{36}{25}}{\frac{89}{138}}\)                                                                       \(\Leftrightarrow B=\frac{\frac{5}{4}}{\frac{2}{5}}\)

\(\Leftrightarrow A=\frac{4968}{2225}\)                                                                      \(\Leftrightarrow B=\frac{25}{8}\)

\(\Leftrightarrow A=\frac{39744}{17800}\)                                                                     \(\Leftrightarrow B=\frac{55625}{17800}\)

Ta có: 39744<55625

\(\Rightarrow A< B\)

Vậy A<B

❤Firei_Star❤
1 tháng 5 2018 lúc 9:34

kb vói mình đã

Quoc Tran Anh Le
Xem chi tiết
Hà Quang Minh
16 tháng 9 2023 lúc 17:08

a) \({( - 2)^4} \cdot {( - 2)^5} = {\left( { - 2} \right)^{4 + 5}} = {\left( { - 2} \right)^9}\)

 \({( - 2)^{12}}:{( - 2)^3} = {\left( { - 2} \right)^{12 - 3}} = {\left( { - 2} \right)^9}\)

Vậy \({( - 2)^4} \cdot {( - 2)^5}\) = \({( - 2)^{12}}:{( - 2)^3}\);

b) \({\left( {\frac{1}{2}} \right)^2} \cdot {\left( {\frac{1}{2}} \right)^6} = {\left( {\frac{1}{2}} \right)^{2 + 6}} = {\left( {\frac{1}{2}} \right)^8}\)

\({\left[ {{{\left( {\frac{1}{2}} \right)}^4}} \right]^2} = {\left( {\frac{1}{2}} \right)^{4.2}} = {\left( {\frac{1}{2}} \right)^8}\)

Vậy \({\left( {\frac{1}{2}} \right)^2} \cdot {\left( {\frac{1}{2}} \right)^6}\) = \({\left[ {{{\left( {\frac{1}{2}} \right)}^4}} \right]^2}\)

c) \({(0,3)^8}:{(0,3)^2} = {\left( {0,3} \right)^{8 - 2}} = {\left( {0,3} \right)^6}\)

\({\left[ {{{(0,3)}^2}} \right]^3} = {\left( {0,3} \right)^{2.3}} = {\left( {0,3} \right)^6}\)

Vậy \({(0,3)^8}:{(0,3)^2}\)= \({\left[ {{{(0,3)}^2}} \right]^3}\).

d) \({\left( { - \frac{3}{2}} \right)^5}:{\left( { - \frac{3}{2}} \right)^3} = {\left( { - \frac{3}{2}} \right)^{5 - 3}} = {\left( { - \frac{3}{2}} \right)^2} = {\left( {\frac{3}{2}} \right)^2}\)

Vậy \({\left( { - \frac{3}{2}} \right)^5}:{\left( { - \frac{3}{2}} \right)^3}\) = \({\left( {\frac{3}{2}} \right)^2}\).

Nguyễn Lê Phước Thịnh
16 tháng 9 2023 lúc 17:10

loading...  loading...  

trangcat25
14 tháng 12 2023 lúc 19:30

(-2) ^4 . (-2) 65 và ( -2) ^ 12 : ( -2) ^3

=( -2) ^ 4+5 =(-2)^9 và (-2) ^12-3 = ( -2) ^9 

vậy ( -2) ^9 = (-2) ^9 

Nên (-2) ^4 .( -2) ^5 = ( -2) ^ 12 : ( -2) ^3

nguyen thanh huyen
Xem chi tiết
❤Firei_Star❤
Xem chi tiết
Nguyễn Mạc Diệu Linh
24 tháng 4 2018 lúc 17:02

A =\(\frac{\left(\frac{17}{5}+\frac{1}{5}\right).\frac{2}{5}}{\left(\frac{38}{7}-\frac{9}{4}\right).\frac{56}{267}}\)

A=\(\frac{36}{25}\).\(\frac{3}{2}\)=\(\frac{54}{25}\)=2,16

B=\(\frac{1,2:\left(\frac{6}{5}-\frac{5}{4}\right)}{0,32+\frac{2}{25}}\)=-24.\(\frac{5}{2}\)=-60

vì 2,16 > -60 Vậy A>B

phạm khánh ly
Xem chi tiết
Isolde Moria
16 tháng 8 2016 lúc 9:26

Ta có

\(A=\frac{\left(1^2-2^2\right)\left(1^2-3^2\right).....\left(1^2-2014^2\right)}{\left(2.3.4.....2014\right)\left(2.3....2014\right)}\)

\(\Leftrightarrow A=\frac{\left(-1\right)3\left(-2\right)4.....\left(-2013\right)2015}{\left(2.3.4.....2014\right)\left(2.3....2014\right)}\)

\(\Leftrightarrow A=\frac{\left[\left(-1\right)\left(-2\right)...\left(-2013\right)\right]\left(3.4.5...2015\right)}{\left(2.3.4.....2014\right)\left(2.3....2014\right)}\)

\(\Leftrightarrow A=\frac{\left(-1\right)2015}{2014.2}=-\frac{2015}{4028}< -\frac{2014}{4028}=-\frac{1}{2}\)

=> A<-1/2